1950 AHSME Problems/Problem 37

Revision as of 13:42, 18 January 2012 by Mrdavid445 (talk | contribs) (Created page with "==Probelm== If <math> y \equal{} \log_{a}{x}</math>, <math> a > 1</math>, which of the following statements is incorrect? <math>\textbf{(A)}\ \text{If }x=1,y=0 \qquad\\ \textbf...")
(diff) ← Older revision | Latest revision (diff) | Newer revision → (diff)

Probelm

If $y \equal{} \log_{a}{x}$ (Error compiling LaTeX. Unknown error_msg), $a > 1$, which of the following statements is incorrect?

$\textbf{(A)}\ \text{If }x=1,y=0 \qquad\\ \textbf{(B)}\ \text{If }x=a,y=1 \qquad\\ \textbf{(C)}\ \text{If }x=-1,y\text{ is imaginary (complex)} \qquad\\ \textbf{(D)}\ \text{If }0<x<z,y\text{ is always less than 0 and decreases without limit as }x\text{ approaches zero} \qquad\\ \textbf{(E)}\  \text{Only some of the above statements are correct}$